Physics for Scientists and Engineers: A Strategic Approach with Modern Physics (3rd Edition)

Published by Pearson
ISBN 10: 0321740904
ISBN 13: 978-0-32174-090-8

Chapter 34 - Electromagnetic Fields and Waves - Exercises and Problems - Page 1029: 4

Answer

${\bf16.3}^\circ\tag{CCW from $+x$-direction}$

Work Step by Step

According to the Galilean fields transformation equations, $$\left.\begin{matrix} &\vec E_B=\vec E_A+\vec v_{BA}\times \vec B_A \\ & \vec B_B=\vec B_A-\dfrac{\vec v_{BA}\times \vec E_A}{c^2}\\ \end{matrix}\right\}\tag 1$$ We are given that - $B_A=(0.5\;\hat k)\;\rm T$, - $E_A=(10^6\cos45^\circ\;\hat i+10^6\sin45^\circ\;\hat j)\;\rm V/m$, - $\vec v_{BA}=(10^6\hat i)\;\rm m/s$ To find $\vec E_B$, we can use the first Galilean's fields transformation equation equation above. $$\vec E_B=\vec E_A+\vec v_{BA}\times \vec B_A $$ Plug the known; $$\vec E_B=(10^6\cos45^\circ\;\hat i+10^6\sin45^\circ\;\hat j)+(10^6\hat i)\times (0.5\;\hat k) $$ $$\vec E_B=(10^6\cos45^\circ\;\hat i+10^6\sin45^\circ\;\hat j)- (0.5\times 10^6\;\hat j) $$ $$\vec E_B=(10^6\cos45^\circ\;\hat i+10^6\sin45^\circ\;\hat j)- (0.5\times 10^6\;\hat j) $$ $$\vec E_B=( 7.07\times 10^5 \;\hat i+2.07\times 10^5 \;\hat j) \;\rm V/m$$ So its angle is given by $$\theta=\tan^{-1}\left[\dfrac{2.07\times 10^5}{ 7.07\times 10^5}\right]$$ $$\theta=\color{red}{\bf16.3}^\circ\tag{CCW from $+x$-direction}$$
Update this answer!

You can help us out by revising, improving and updating this answer.

Update this answer

After you claim an answer you’ll have 24 hours to send in a draft. An editor will review the submission and either publish your submission or provide feedback.